Đến nội dung

Hình ảnh

Tổng hợp các bài BĐT

ẩn đi không cho xem

  • Please log in to reply
Chủ đề này có 164 trả lời

#101
Dung Gia

Dung Gia

    Binh nhất

  • Thành viên
  • 22 Bài viết

cho em hỏi bài này ạ : Cho $a,b,c\geq 0;a+b+c+ab+ac+bc=3. CMR:a^{4} +b^{4}+c^{2}\geq2$


Bài viết đã được chỉnh sửa nội dung bởi Dung Gia: 27-07-2016 - 20:04


#102
Olivier Gooner

Olivier Gooner

    Lính mới

  • Thành viên mới
  • 2 Bài viết

Cho $a,b,c$ là các số thực dương. Chứng minh rằng : $\frac{1}{x(y+1)}+\frac{1}{y(z+1)}+\frac{1}{z(x+1)}\geq \frac{3}{xyz+1}$



#103
Olivier Gooner

Olivier Gooner

    Lính mới

  • Thành viên mới
  • 2 Bài viết

Cho $a,b,c>0$ và $a+b+c=6$. Tìm giá trị nhỏ nhất của biểu thức:

 

$A=\frac{\sqrt{a^2+ab+b^2}}{bc+4}+\frac{\sqrt{b^2+bc+c^2}}{ca+4}+\frac{\sqrt{c^2+ac+a^2}}{ab+4}$



#104
Hoangtheson2611

Hoangtheson2611

    Sĩ quan

  • Thành viên
  • 435 Bài viết

Bài 1 : Cho a,b,c>0 ; a+b+c=1 . Tìm Max của $\sum \frac{a-bc}{a+bc}$

Bài 2 : Cho a,b,c>0 ; $\sum a^{2}+abc=4$ . CMR : $\sum \frac{bc}{a}\geqslant \sum a^{2}$



#105
Air Force

Air Force

    Trung sĩ

  • Thành viên
  • 145 Bài viết

Bài 1:

Cho abc=1 va $a^{3}> 36.  CMR  :\frac{a^{2}}{3}+b^{2}+c^{2}> ab +bc+ca$}

Lời giải:

$VT-VP=\frac{a^{2}}{4}+b^{2}+c^{2}-ab-bc+2bc+\frac{a^{2}}{12}=(\frac{a}{2}-b-c)^{2}+\frac{a^{2}-36bc}{12}>0\Rightarrow$ đpcm

Cách khác:

Từ giả thiết suy ra $a>0$ và $bc>0$. Bất đẳng thức cần chứng minh tương đương với
\[\dfrac{a^2}{3}+(b+c)^2-3bc-a(b+c)\ge 0\\ \iff \dfrac{1}{3}+\left(\dfrac{b+c}{a}\right)^2-\dfrac{b+c}{a}-\dfrac{3}{a^3}\ge 0\]
Vì $a^3>36$ nên \[\dfrac{1}{3}+\left(\dfrac{b+c}{a}\right)^2-\dfrac{b+c}{a}-\dfrac{3}{a^3}> \left(\dfrac{b+c}{a}\right)^2-\dfrac{b+c}{a}+ \dfrac{1}{4}= \left(\dfrac{b+c}{a}-\dfrac{1}{2}\right)^2 >0\]
 
 

 

Bài 2:

Với a,b,c >0; n ∈ N*.CMR:

$\frac{a^{n}}{b+c}+\frac{b^{n}}{a+c}+\frac{c^{n}}{a+b}\geq \frac{3}{2}\left ( \frac{a^{n}+b^{n}+c^{n}}{a+b+c} \right )$

Lời giải:

$\sum \frac{a^{n}}{b+c}\geq \frac{1}{3}(\sum a^{n})(\sum \frac{1}{a+b})\geq \frac{1}{3}(\sum a^{n})(\frac{9}{2(a+b+c)})=\frac{3}{2}(\frac{\sum a^{n}}{\sum a})$

 

 

 

Bài 3:

Cho $x,y,z >0$ thỏa điều kiện $x^{2}+y^{2}+z^{2}=9$

Tìm giá trị nhỏ nhất của $P=\frac{x^5}{y^2}+\frac{y^5}{z^2}+\frac{z^5}{x^2}$

Lời giải:

Theo $Cauchy$ Ta có:

$$\dfrac{x^5}{y^2}+\dfrac{x^5}{y^2}+\sqrt{3}y^2+\sqrt{3}y^2+3\sqrt{3}\ge \sqrt{3}x^2$$

Cách khác:

Sử dụng Cauchy-Schwarzt ta có 

       $\frac{x^5}{y^2}+\frac{y^5}{z^2}+\frac{z^5}{x^2}\geqslant \frac{(x^3+y^3+z^3)^2}{xy^2+yz^2+zx^2}$

Sử dụng Cauchy-Schwarzt và AM-GM ta có 

       $xy^2+yz^2+zx^2\leqslant \sqrt{(x^2+y^2+z^2)(x^2y^2+y^2z^2+z^2x^2)}\leqslant \sqrt{\frac{(x^2+y^2+z^2)^3}{3}}=3$

Do đó $P\geqslant \frac{(x^3+y^3+z^3)^2}{3}\geqslant \frac{(x^2+y^2+z^2)^3}{9}=3$ 

Đẳng thức xảy ra khi $x=y=z=1$

 

 

 

Bài 4:

Ch0 $a>0$ và $n$ là 1 số tự nhiên

Chứng minh rằng $a^n+\frac{1}{a^n}-2\geqslant n^2(a+\frac{1}{a}-2)$

Lời giải:

Bất đẳng thức tương đương với $(a^{n-1}+a^{n-2}+...+a+1)\geq n^2a^{n-1}$ (hiển nhiên theo AM-GM)

Cách khác:

Do tính đối xứng giữa a và $\frac{1}{a}$ nên ta có thể giả sử a ≥ 1.  đặt $\sqrt{a}$ =x ≥ 1.bdt $\Leftrightarrow$ $x^{2n}+\frac{1}{x^{2n}}-2 \geq n^{2}(x^{2}+\frac{1}{x^{2}}-2)\Leftrightarrow (x^{n}-\frac{1}{x^{n}})^{2}\geq n^{2}(x-\frac{1}{x})^{2} \Leftrightarrow $x^{n}-\frac{1}{x^{n}}\geq n(x-\frac{1}{x})$①.

Với x=1 thì ① đúng

Với x>1 thì ① $\Leftrightarrow x^{n-1} +x^{n-3} ...+\frac{1}{x^{n-3}}+\frac{1}{x^{n-1}}\geq n$ (đúng vì theo bđt AM-GM).

Dấu bằng xảy ra khi x=1 $\Leftrightarrow a=1$

 

 

 

Bài 5:

Cho $a,b,c,d$ là các số thực thỏa mãn $\left\{\begin{matrix} a+b+c+d=0\\a^2+b^2+c^2+d^2=2 \end{matrix}\right.$

Tìm GTLN của $P=abcd$

Lời giải:

Áp dụng AM-GM ta có 

$2=\sum a^{2}\geq 4\sqrt[4]{\prod a^{2}}\Rightarrow \sqrt{\left | abcd \right |}\leq \frac{1}{2}\Rightarrow abcd\leq \frac{1}{4}$

Dấu bằng xảy ra khi $a=b=-c=-d=\frac{1}{\sqrt{2}}$ và các hoán vị của chúng

 

 

 

Bài 6:

Cho $a,\,b,\,c\geq 0$ thỏa mãn $a+b+c=1.$ Tìm giá trị lớn nhất của biểu thức: $$P=abc\left(a^2+b^2+c^2\right)$$

Lời giải:

Ta có: $P=abc(a+b+c)(a^{2}+b^{2}+c^{2})\leq \frac{1}{3}(ab+bc+ca)^{2}(a^{2}+b^{2}+c^{2})$

 

Mặt khác, lại có: $(ab+bc+ca)^{2}(a^{2}+b^{2}+c^{2})\leq \left (\frac{(a+b+c)^{2}}{3} \right )^{3}=\frac{1}{27}$

 

Do đó: $P\leq \frac{1}{81}$

 

Dấu bằng xảy ra khi và chỉ khi $a=b=c= \frac{1}{3}$

 

 

 

Bài 7:

Cho các số thực $x,\,y>0$ thỏa mãn $3x+y\leq1.$ Tìm giá trị nhỏ nhất của biểu thức: $$S=\dfrac{1}{x}+\dfrac{1}{\sqrt{xy}}$$

Lời giải:

$S\geq \frac{1}{x}+\frac{1}{\sqrt{x(1-3x)}}$

$\geq \frac{1}{x}+\frac{2}{1-2x}=\frac{2}{x(1-x)}\geq \frac{8}{(x+1-x)^{2}}=8$

Dấu "=" xảy ra $\Leftrightarrow x=\frac{1}{4}$

 

 

 

Bài 8:

Cho các số thực a,b,c,x,y thỏa mãn $$ax-by=\sqrt{3}$$ .

Tìm GTNN của $F= a^{2}+b^{2}+x^{2}+y^{2}+ bx +ay$

Lời giải:

Sử dụng giả thiết $ax-by=\sqrt{3}$ ta có:
$$(a^2+b^2)(x^2+y^2)=(ax+by)^2+(ax-by)^2=(ax+by)^2+3$$
Áp dụng bất đẳng thức $Cauchy$ , suy ra:
$$a^2+b^2=x^2+y^2=(a^2+b^2)+(x^2+y^2) \\ \ge 2\sqrt{(a^2+b^2)(x^2+y^2)}=2\sqrt{(ax+by)^2+3}$$
Do đó, ta đưa về bài toán tìm GTNN của: $2\sqrt{x^2+3}+x$ trong đó $x=ax+by$
Ta có:
$$\left(2\sqrt{x^2+3}+x\right)^2=4(x^2+3)+4x\sqrt{x^2+3}+x^2 \\ = (x^2+3)+4x\sqrt{x^2+3}+4x^2+9 \\ = \left(\sqrt{x^2+3}+2x\right)^2+9\ge 9$$
$$\Rightarrow 2\sqrt{x^2+3}+x\ge 3$$
Vậy $\text{MinT}=\fbox{3}$
 
 
 

Bài 9:

Cho các số thực dương $a,b,c$. Tìm giá trị nhỏ nhất của biểu thức:

 

                                $P=\frac{2}{a+ \sqrt{ab}+ \sqrt[3]{abc}}-\frac{3}{\sqrt{a+b+c}}$

Lời giải:

$a+\sqrt{\frac{1}{2}a.2b}+\sqrt[3]{\frac{1}{4}a.b.4c}\leq a+\frac{1}{4}a+b+\frac{1}{12}a+\frac{1}{3}b+\frac{4}{3}c=\frac{4}{3}(a+b+c)$

Do đó $P\geq \frac{3}{2(a+b+c)}-\frac{3}{\sqrt{a+b+c}}$...

 

 

 

Bài 10:

Cho x,y là các số không âm thoả $x^{3}+y^{3}\leq 1$

 

Tìm giá trị lớn nhất của $P=2\sqrt{x}+\sqrt{y}$

Lời giải:

$(x^3+y^3)(\sqrt[5]{2^6}+1)^5\geqslant (2\sqrt{x}+\sqrt{y})^6$

$\Leftrightarrow 2\sqrt{x}+\sqrt{y}\leqslant \sqrt[6]{(\sqrt[5]{2^6}+1)^5}$

Vậy $Max(P)= \sqrt[6]{(\sqrt[5]{2^6}+1)^5}\Leftrightarrow \frac{a^3}{2\sqrt[5]{2}}=b^3=\frac{1}{2\sqrt[5]{2}+1}$

 

Chiều coppy tiếp, sau đó kiểm tra nội dung sau.

 

 

Bài 11:Cho các số a,b,c không âm không đồng thời bằng không. Chứng minh rằng;

$\sum \frac{2a^{2}-bc}{b^{2}-bc+c^{2}}\geq 3$

 

Lời giải:(vutuanhien)

 

Không mất tính tổng quát, ta có thể giả sử $b$ là số nằm giữa $a$ và $c$

BĐT đã cho tương đương với

$$\sum \frac{2a^2+(b-c)^2}{b^2-bc+c^2}\geq 6$$

Áp dụng BĐT Cauchy-Schwarz, ta có

$$\sum \frac{2a^2}{b^2-bc+c^2}\geq \frac{2(a^2+b^2+c^2)^2}{\sum a^2(b^2-bc+c^2)}=\frac{2(a^2+b^2+c^2)^2}{2\sum a^2b^2-abc\sum a}$$

$$\sum \frac{(b-c)^2}{b^2-bc+c^2}\geq \frac{[a(b-c)+b(a-c)+c(a-b)]^2}{2\sum a^2b^2-abc\sum a}=\frac{4b^2(a-c)^2}{2\sum a^2b^2-abc\sum a}$$

Do đó ta chỉ cần chứng minh

$$(a^2+b^2+c^2)^2+2b^2(a-c)^2\geq 6\sum a^2b^2-3abc\sum a (1)$$

Ta có 

$b^2(a-c)^2=[a(b-c)+c(a-b)]^2=a^2(b-c)^2+c^2(a-b)^2+2ac(a-b)(b-c)$

$\geq a^2(b-c)^2+c^2(a-b)^2$

Suy ra 

$$2b^2(a-c)^2\geq a^2(b-c)^2+b^2(c-a)^2+c^2(a-b)^2$$

$$\Rightarrow VT (1)\geq (\sum a^2)^2+2\sum a^2b^2-2abc\sum a$$

Do đó ta chỉ còn phải chứng minh 

$$(\sum a^2)^2+2\sum a^2b^2-2abc\sum a\geq 6\sum a^2b^2-3abc\sum a$$

$$\Leftrightarrow \sum a^4+abc\sum a\geq 2\sum a^2b^2$$

BĐT này hiển nhiên đúng theo BĐT Schur

$$\sum a^4+abc\sum a\geq \sum ab(a^2+b^2)$$

Và BĐT AM-GM

$$\sum ab(a^2+b^2)\geq 2\sum a^2b^2$$

Kết thúc chứng minh 

Đẳng thức xảy ra khi $a=b=c$ hoặc $a=b$, $c=0$ và các hoán vị.

 

Bài 12:(bosulan239)

Cho a,b,c là các số không âm không đồng thời bằng không.

CMR

$\frac{\sum a^{2}}{\sum ab}\geq \sum \frac{ab}{b^{2}+bc+c^{2}}$

Bài giải:(vutuanhien)

 

BĐT đã cho tương đương với

$\frac{a^2}{ab+bc+ca}-\frac{ab}{b^2+bc+c^2}+\frac{b^2}{ab+bc+ca}-\frac{bc}{c^2+ca+a^2}+\frac{c^2}{ab+bc+ca}-\frac{ca}{a^2+ab+b^2}\geq 0$

$\Leftrightarrow \sum \frac{ac(ac-b^2)}{b^2+bc+c^2}\geq 0$

Do $\frac{ac(ac-b^2)}{b^2+bc+c^2}=\frac{ac^2(a+b+c)}{b^2+bc+c^2}-ac$ nên BĐT đã cho có thể viết lại thành

$\sum \frac{ac^2(a+b+c)}{b^2+bc+c^2}\geq ab+bc+ca$

$\Leftrightarrow \sum \frac{ac^2}{b^2+bc+c^2}\geq \frac{ab+bc+ca}{a+b+c}$

Áp dụng BĐT Cauchy-Schwarz, ta có

$VT\geq \frac{(ab+bc+ca)^2}{\sum a(b^2+bc+c^2)}=\frac{ab+bc+ca}{a+b+c}$

Kết thúc chứng minh 

 

 


Bài 13: (nguyencuong123)

Cho a,b,c không âm thoả mãn: $a+b+c=3$

Chứng Minh: $\sum \frac{a+1}{ab+1}\geq 3$

 

 

Bài giải:(Juliel)

Áp dụng AM-GM cho vế trái, ta cần chứng minh :

$(a+1)(b+1)(c+1)\geq (ab+1)(bc+1)(ca+1)\Leftrightarrow abc+(ab+bc+ca)+(a+b+c)+1\geq a^{2}b^{2}c^{2}+abc(a+b+c)+(ab+bc+ca)+1\Leftrightarrow abc+4\geq a^{2}b^{2}c^{2}+3abc+1\Leftrightarrow a^{2}b^{2}c^{2}+2abc\leq 3$

Hiển nhiên đúng vì $abc\leq (\frac{a+b+c}{3})^{3}=1$

 


 


Bài 14:(Chrome98):Chứng minh bất đẳng thức sau với $a,b,c>0$ và $a+b+c=1$:

 

\[ \frac{a^2}{3a+1}+\frac{b^2}{3b+1}+\frac{c^2}{3c+1}\ge 24\left(\frac{a^2}{9a+1}+\frac{b^2}{9b+1}+\frac{c^2}{9c+1}\right)^2 \]

Bài giải: (Simpson Joe Donald)

 

 
 
$\bullet\ AM-GM:\ \dfrac{a^2}{9a+1}= \dfrac{a^2}{6a+(3a+1)}\le \dfrac{a^2}{2\sqrt{6a(3a+1)}}= \dfrac{a\sqrt{a}}{2\sqrt{6(3a+1)}}$ ;
$\bullet\ Cauchy-Schwarz:\ VP\le \left( \dfrac{a\sqrt{a}}{\sqrt{3a+1}}+  \dfrac{b\sqrt{b}}{\sqrt{3b+1}}+  \dfrac{c\sqrt{c}}{\sqrt{3c+1}}\right)^2\le (a+b+c).VT=VT$

 

Bài 15:(trauvang97:)Cho các số thực dương $a,b,c$ thoả mãn:

 

                             $\frac{2}{a^{2}+1}+\frac{2}{b^{2}+1}+\frac{2}{c^{2}+1}\geq 3$

 

Chứng minh rằng: $(a-2)^{2}+(b-2)^{2}+(c-2)^{2}\geq 3$

 

Bài giải:

 

(Nguyen Huy Tuyen)$\frac{2}{a^{2}+1}+\frac{2}{b^{2}+1}+\frac{2}{c^{2}+1}\geq 3\Leftrightarrow \sum \frac{(1-a)(1+a)}{a^2+1}\geqslant 0$

$(a-2)^{2}+(b-2)^{2}+(c-2)^{2}-3=\sum (a-3)(a-1)$

Ta có :$\sum (a-3)(a-1)-\sum \frac{2(1-a)(1+a)}{a^2+1}=\sum \frac{(a-1)^4}{a^2+1}\geqslant 0$

           $\Leftrightarrow \sum (a-3)(a-1)\geqslant \sum \frac{2(1-a)(1+a)}{a^2+1}\geqslant 0$

           $\Leftrightarrow (a-2)^{2}+(b-2)^{2}+(c-2)^{2}\geq 3$

 

 

Bài 16:(phanquockhanh)Cho $x,y,z >0 : xyz+x+z=y$ . Tìm giá trị lớn nhất của biểu thức : $P=\frac{2}{x^2+1} - \frac{2}{y^2+1} -\frac{4z}{\sqrt{z^2+1}}+\frac{3z}{(z^2+1).\sqrt{z^2+1}}$

(Trích đề thi thử số 2 – THTT)

Bài giải:

(trauvang97)Từ giả thiết ta có: $x=\frac{y-z}{1+yz}$.

 

Khi đó:

 

$P=\frac{2(1+yz)^{2}}{(y^{2}+1)(z^{2}+1)}-\frac{2}{y^{2}+1}-\frac{4z}{\sqrt{z^{2}+1}}+\frac{3z}{(z^{2}+1)\sqrt{z^{2}+1}}$

 

$P=\frac{2z(2y+(y^{2}-1)z)}{(y^{2}+1)(z^{2}+1)}-\frac{4z}{\sqrt{z^{2}+1}}+\frac{3z}{(z^{2}+1)\sqrt{z^{2}+1}}$

 

Do $\frac{2z(2y+(y^{2}-1)z)}{(y^{2}+1)(z^{2}+1)}=\frac{2z\sqrt{(2y+(y^{2}-1)z)^{2}}}{(y^{2}+1)(z^{2}+1)}\leq \frac{2z\sqrt{(4y^{2}+(y^{2}-1)^{2})(1+z^{2})}}{(y^{2}+1)(z^{2}+1)}=\frac{2z}{\sqrt{z^{2}+1}}$

 

Do đó:

 

$P\leq \frac{2z}{\sqrt{z^{2}+1}}-\frac{4z}{\sqrt{z^{2}+1}}+\frac{3z}{\sqrt{z^{2}+1}}\left ( 1-\frac{z^{2}}{z^{2}+1} \right )$

 

$P=-3t^{3}+t$ với $\frac{z}{\sqrt{z^{2}+1}}=t\in (0;1)$

 

Khảo sát hàm số trên ta thấy $maxP=\frac{2}{9}\Leftrightarrow x=\frac{\sqrt{2}}{2};y=\sqrt{2},z=\frac{\sqrt{2}}{4}$

 

 

 


Bài 17:(Toc Ngan)Cho $a,b,c >0$ và $a+b+c=3$

Chứng minh rằng : $8(\frac{1}{a}+\frac{1}{b}+\frac{1}{c})+9 \geq 10(a^2+b^2+c^2)$

 Bài giải:(babystudymaths)

 

Cách 1

Giải như sau:

Giả sử a là số lớn nhất trong 3 số a,b,c ,thế thì c nhỏ hơn 3 và không nhỏ hơn 1

ta thấy $9=(42a-48)+(42b-\frac{69}{2})+(42c-\frac{69}{2})$

Thay và BĐT ban đầu ta thấy tương đương

$(\frac{8}{b}-10b^{2}+42b-\frac{69}{2})+(\frac{8}{c}-10c^{2}+42c-\frac{69}{2})\geq 10a^{2}-\frac{8}{a}-42a+48\Leftrightarrow \frac{(16-5b)(2b-1)^{2}}{b}+\frac{(16-52)(2c-1)^{2}}{c}\geq \frac{4(5a-1)(a-2)^{2}}{a}$

Áp dụng BCS ,ta có:

VT $\geq \frac{(2b-1+2c-1)^{2}}{\frac{b}{16-5b}+\frac{c}{16-5c}}= \frac{4(a-2)^{2}}{\frac{b}{16-5b}+\frac{c}{16-5c}}$

Lúc này ta chỉ cần chứng minh 

$\frac{a}{5a-1}\geq \frac{b}{16-5b}+\frac{c}{16-5c}$

Mà $\frac{b}{16-5b}+\frac{c}{16-5c}\leq \frac{b}{16-5a}+\frac{c}{16-5a}= \frac{3-a}{16-5a}\leq \frac{a}{5a-1}\Leftrightarrow \frac{1}{(5a-1)(16-5a)}> 0$

ĐÚng theo giả thiết,từ đây ta suy ra đ.p.c.m

Đẳng thức xảy ra khi và chỉ khi a=b=1/2 ,c=2 cùng hoán vị

 

Cách 2: 

Ta có BĐT tương đương

f(abc,a+b+c,ab+bc+ca) =$8.\frac{ab+bc+ca}{abc}+9-10((a+b+c)^{2}-2(ab+bc+ca))\geq 0$

Nhận thấy đây là hàm đơn điệu trên R theo abc nên theo định lý ABC, hàm số đạt cực tiểu khi có 2 biến = nhau, nên a=b=$\frac{3-c}{2}$

Thay vào và chứng minh BĐT 1 biến c ,bài toán trở nên quá đơn giản 

 

 

 

 


Bài 18:(caybutbixanh)Cho $x;y;z> 0$.Chứng minh rằng :

$P=\frac{2xy}{(z+x)(z+y)}+\frac{2yz}{(x+y)(x+z)}+\frac{3xz}{(y+z)(y+x)}\geqslant \frac{5}{3}.$

 

(trích đề thi học sinh giỏi lớp 11-Quảng Bình 2011)

--------------------------------

 

 

 

(T M) Hướng giải:

 

Bằng khai triển trực tiếp ta đưa bất đẳng thức cần chứng minh thành

 

$$xy(x+y)+yz(y+z)+4xz\left ( x+z \right )\geq 10xyz$$

 

Điều này tương đương với

 

$$\frac{x+y}{z}+\frac{y+z}{x}+\frac{4(x+z)}{y}\geq 10$$

 

Áp dụng $AM-GM$ từng cặp là ra.

 

 


Bài 19:(supermath98)Cho các sô dương $a;b;c$ thỏa mãn $\large ab+ac+bc=3abc$. Tìm GTNN của biểu thức: 

 

$\large M=\frac{2\left ( a^{2}b^{2}+b^{2}c^{2}+a^{2}c^{2} \right )+abc}{a^{2}b^{2}c^{2}}$

 

 

Bài giải:(thanhdok14)

 

 

Vì $a, b, c>0$ nên từ điều kiện ban đầu, ta suy ra:

$\frac{1}{a}+\frac{1}{b}+\frac{1}{c}=3$

Đặt: $\left(\frac{1}{a};\frac{1}{b};\frac{1}{c}\right)\to (x;y;z)$

$\Rightarrow x+y+z=3$

$\Rightarrow xy+yz+zx\le 3$

Mặt khác: $M$ được viết lại thành:

$M=2\left(\frac{1}{a^2}+\frac{1}{b^2}+\frac{1}{c^2}\right)+\frac{1}{abc}$

$=2(x^2+y^2+z^2)+xyz$

Lại có: $x^2+y^2+z^2=9-2(xy+yz+zx)$

$xyz\ge \frac{(x+y+z)[4(xy+yz+zx)-(x+y+z)^2]}{9}=\frac{4(xy+yz+zx)-9}{3}$   (theo $schur$)

Từ đó ta có:

$M\ge \frac{4}{3}(xy+yz+zx)-4(xy+yz+zx)+15=\frac{-8}{3}(xy+yz+zx)+15\ge 7$   (vì $xy+yz+zx\le 3$)

Vậy $min M=7\Leftrightarrow a=b=c=1$

 

 


Bài 20:(duaconcuachua)

Cho $a,b,c$ là các số thực dương thỏa mãn $ab+bc+ca=abc$.

Chứng minh rằng $\frac{a^{4}+b^{4}}{ab(a^{3}+b^{3})}+\frac{b^{4}+c^{4}}{bc(b^{3}+c^{3})}+\frac{c^{4}+a^{4}}{ca(c^{3}+a^{3})}\geq 1$

 Bài giải:(Sagittius912)Theo bđt Chebyshev ta có

 

 

$\frac{a^4+b^4}{a^3+b^3}\ge \frac{a+b}{2}$

do đó

 

$\frac{a^{4}+b^{4}}{ab(a^{3}+b^{3})}+\frac{b^{4}+c^{4}}{bc(b^{3}+c^{3})}+\frac{c^{4}+a^{4}}{ca(c^{3}+a^{3})}\ge \frac{a+b}{2ab}+\frac{b+c}{2bc}+\frac{c+a}{2ca}=\frac{ab+bc+ca}{abc}=1$

 

Dấu đẳng thức xảy ra khi $a=b=c=3$

 

 



#106
canletgo

canletgo

    Sĩ quan

  • Thành viên
  • 389 Bài viết

Mọi người giúp mình bài này với ​ !!!  :icon6:  :icon6:  :icon6:

Cho a, b, x, y $\in R^{+}$ thỏa mãn:

                                                                        a + b = 1

                                                                        ax + by = 2

                                                                     $ax^{2} + by^{2} = 3$

CMR :            $4 < ax^{3} + by^{3} < 4,5$


Bài viết đã được chỉnh sửa nội dung bởi canletgo: 22-08-2016 - 17:05

Alpha $\alpha$ 


#107
babykai

babykai

    Lính mới

  • Thành viên mới
  • 9 Bài viết

Cho $a, b, c > 0$. Chứng minh :

$\frac{a^{3}}{b^{2}} + \frac{b^{3}}{c^{2}} + \frac{c^{3}}{a^{2}} \geq \frac{a^{2}}{b} + \frac{b^{2}}{c} + \frac{c^{2}}{a}$

Đầu tiên dễ chứng minh được a2+ b2+ c2 ≥ ab+bc+ca

Ta có (a3)/b+ ab ≥ 2a2
(b3)/c+ bc ≥ 2b2
(c3)/a +ca ≥ 2c2 

(a3)/b + (b3)/c+ (c3)/a + ab+ ac+ bc ≥ 2a2+ 2b2+ 2c2 => đpcm
b) Ta có
2a2+ 2b2+ 2c2 ≥ 2ab+2bc+2ca
a2+ b2+ c2 +3 ≥ 2a+2b+2c
=> 3a2+ 3b2+ 3c2 +3 ≥ 2ab+2bc+2ca +2a+2b+2c =12 => đpcm


My own angel


#108
Kiratran

Kiratran

    Thượng sĩ

  • Thành viên
  • 296 Bài viết

$Cho x,y,z>0, x+y+z=3 . CMR: \frac{xy}{z}+ \frac{yz}{x} + \frac{xz}{y} \geq 3$ 


Bài viết đã được chỉnh sửa nội dung bởi Kiratran: 11-11-2016 - 19:38

Duyên do trời làm vương vấn một đời.


#109
thjiuyghjiuytgjkiutghj

thjiuyghjiuytgjkiutghj

    Hạ sĩ

  • Thành viên
  • 70 Bài viết

$Cho x,y,z>0, x+y+z=3 . CMR: \frac{xy}{z}+ \frac{yz}{x} + \frac{xz}{y} \geq 3$

Với điều kiện $x,y,z>0 (gt)$, áp dụng bất đẳng thức Cauchy ta được :
$\frac{xy}{z} + \frac{yz}{x} \geq 2\sqrt{\frac{xy}{z} . \frac{yz}{x}} = 2\sqrt{y^{2}} = 2y$
$\frac{yz}{x} + \frac{zx}{y} \geq 2\sqrt{\frac{yz}{x} . \frac{zx}{y}} = 2\sqrt{z^{2}} = 2z$
$\frac{zx}{y} + \frac{xy}{z} \geq 2\sqrt{\frac{zx}{y} . \frac{xy}{z}} = 2\sqrt{x^{2}} = 2x$
Cộng vế theo vế ta được : $\frac{xy}{z}+ \frac{yz}{x} + \frac{xz}{y} \geq x+y+z = 3$
Khi $x=1,y=1,z=1$ (thoả yêu cầu đề bài) thì đẳng thức xảy ra.
Vậy $Min_{\frac{xy}{z}+ \frac{yz}{x} + \frac{xz}{y}} = 6$ khi $x=1,y=1,z=1$

Bài viết đã được chỉnh sửa nội dung bởi thjiuyghjiuytgjkiutghj: 19-11-2016 - 22:14


#110
haivana1619

haivana1619

    Binh nhất

  • Thành viên mới
  • 24 Bài viết

mọi người giúp em bài này với ạ:

cho x,y>0. Tìm Min Q=$(3+\frac{1}{x})(3+\frac{1}{y})(2+x+y)$



#111
ChienThanga1k49

ChienThanga1k49

    Lính mới

  • Thành viên mới
  • 5 Bài viết

Cho a,b,c là các số thực dương thỏa mãn ab+bc+ca=1.Chứng minh rằng :



#112
duongduong2406

duongduong2406

    Lính mới

  • Thành viên mới
  • 8 Bài viết

Chứng minh :

b, Nếu $a, b, c \geq -1, a + b + c = 1$ thì : $\frac{a}{1 + a^{2}} + \frac{b}{1 + b^{2}} + \frac{c}{1 + c^{2}} \leq \frac{9}{10}$

câu b như nào vậy bạn



#113
Hanhphuclavay

Hanhphuclavay

    Binh nhất

  • Thành viên
  • 23 Bài viết

cho 3 số dương a,b,c thỏa mãn a+ b+ c=12 . cm 

  1/(a+b) + 1/(b+c) + 1/(c+a) >= 8/(28+a^2)   +    8/(28+b^2)     +      8/(28+c^2)



#114
ghivayra

ghivayra

    Lính mới

  • Thành viên mới
  • 6 Bài viết

mình có bài này khó qúa bạn nào giúp mình với

a,b,c là độ dài ba cạnh tam giác không nhọn chứng minh

(a2 +b2 +c2)(1/a2 + 1/b2 + 1/c2 ) >= 10



#115
NHoang1608

NHoang1608

    Sĩ quan

  • Thành viên
  • 375 Bài viết

Solution:

  Vì $a,b,c$ là 3 cạnh của tam giác không nhọn nên là 3 cạnh của tam giác tù :)

 *Bổ đề: Nếu $x, y, z$ là độ dài ba cạnh của 1 tam giác tù và thỏa mãn $x=max\left \{ x;y;z \right \}$ thì $x^{2}\geq y^{2}+z^{2}$ (Tự c/m  bằng hình học)

 Áp dụng bđt Cauchy-Schwarz thì ta có $\frac{1}{b^{2}}+ \frac{1}{c^{2}} \geq \frac{4}{b^{2}+c^{2}}$.

 Suy ra $P\geq (\frac{1}{a^{2}}+ \frac{4}{b^{2}+c^{2}})(a^{2}+b^{2}+c^{2})$.

                  $=  5 + \frac{b^{2}+c^{2}}{a^{2}}+\frac{4a^{2}}{b^{2}+c^{2}}$  $(*)$

 Ap dụng bđt AM-GM thì $\frac{a^{2}}{b^{2}+c^{2}}+\frac{b^{2}+c^{2}}{a^{2}}\geq 2$  $(1)$

 Mặt khác áp dụng bổ đề thì ta có: $\frac{3a^{2}}{b^{2}+c^{2}}\geq 3$  $(2)$

 Cộng $(1)$ với $(2)$ thì được $\frac{b^{2}+c^{2}}{a^{2}}+\frac{4a^{2}}{b^{2}+c^{2}}\geq 5$  $(3)$

 Từ $(*)$ và $(3)$ thì $P\geq 10$ . Đẳng thức xảy ra khi $a=\sqrt{2}b=\sqrt{2}c$


Bài viết đã được chỉnh sửa nội dung bởi NHoang1608: 11-03-2017 - 21:28

The greatest danger for most of us is not that our aim is too high and we miss it, but that it is too low and we reach it.

----- Michelangelo----


#116
Drago

Drago

    Sĩ quan

  • Thành viên
  • 462 Bài viết

Cho $a,b,c$ là các số thực không âm. Chứng minh rằng:

$1.(a+b+c)^{3} \geq 6\sqrt{3}(a-b)(b-c)(c-a)$

$2.a^{3}+b^{3}+c^{3}-3abc \geq 2(\frac{b+c}{2}-a)^{3}$


Bài viết đã được chỉnh sửa nội dung bởi Drago: 19-05-2017 - 23:54

$\mathbb{VTL}$


#117
canletgo

canletgo

    Sĩ quan

  • Thành viên
  • 389 Bài viết

Với mọi a, b, c không âm. CMR:

$(a+b)(b+c)(c+a)\geq \frac{8}{9}(a+b+c)(\frac{a}{b}+\frac{b}{c}+\frac{c}{a})$


Alpha $\alpha$ 


#118
kuhaza

kuhaza

    Trung sĩ

  • Thành viên
  • 102 Bài viết

cho a ; b ; c là các số thực dương thoả mãn a+b+c=3 cmr

$\sqrt{\frac{a}{3b^{2}+1}}+\sqrt{\frac{b}{3c^{2}+1}}+\sqrt{\frac{c}{3a^{2}+1}}\geq \frac{3}{2}$


Bài viết đã được chỉnh sửa nội dung bởi kuhaza: 09-09-2017 - 21:41


#119
lengocduc195

lengocduc195

    Binh nhì

  • Thành viên mới
  • 13 Bài viết

Em không biết cách gõ nên bài đầu tiên em gửi đến như thế này. Xin mọi người thông cảm ạ

24.jpg


Đời là cát bụi  :lol:  :lol:  :lol:


#120
DOTOANNANG

DOTOANNANG

    Đại úy

  • ĐHV Toán Cao cấp
  • 1609 Bài viết

$$ x,y,z > 0$$ $$ x+y+z=3$$

$$ (x^2y+y^2z+z^2x)\left[\frac{1}{(x+y)^2}+\frac{1}{(y+z)^2} +\frac{1}{(z+x)^2}\right] \geq  \frac94$$






0 người đang xem chủ đề

0 thành viên, 0 khách, 0 thành viên ẩn danh